If $f$ is continuous $displaystylelim_{x to infty}f(x)=infty$ iff $f^{-1}(K)$ is compact for every compact $K...












1












$begingroup$



Let $f: mathbb{R}^{m}to mathbb{R}^{n}$ be a continuous function. Prove that the following statements are equivalents:



(i) $displaystylelim_{x to infty}f(x)=infty$;



(ii) $f^{-1}(K)$ is compact for every compact $K subset mathbb{R}^{n}$.




My attempt.



(i) $Longrightarrow$ (ii): If $K subset mathbb{R}^{n}$ is compact, $f^{-1}(K)$ is closed, since $f$ is continuous. If $f^{-1}(K)$ is unbounded, there is a sequence $(x_{n}) subset f^{-1}(K)$ such that $x_{n} to infty$.



I want to conclude that $f(x_{n}) to infty$. How can I ensure it?



(ii) $Longrightarrow$ (i): I dont have a good idea. Can someone help me? I want just a hint, not a solution.










share|cite|improve this question









$endgroup$












  • $begingroup$
    How do you define $xtoinfty$ exactly? With the one-point compactification?
    $endgroup$
    – SmileyCraft
    Jan 6 at 3:55








  • 1




    $begingroup$
    Consider the function $f(x)=e^x$ and the compact set $K=[-1,1]$. Then $lim f(x)=infty$ but $f^{-1}(K)=(-infty,0]$. So either this problem isn't correct or I am losing my mind.
    $endgroup$
    – ItsJustLogicBro
    Jan 6 at 3:58












  • $begingroup$
    @SmileyCraft, my book has not yet exactly defined $x to infty$. When we work with infinite limits, we use the standard definition.
    $endgroup$
    – Lucas Corrêa
    Jan 6 at 4:02










  • $begingroup$
    @ItsJustLogicBro $limlimits_{x to -infty } e^x =0$. So this isn’t a counterexample.
    $endgroup$
    – mathcounterexamples.net
    Jan 6 at 4:02






  • 1




    $begingroup$
    @LucasCorrêa I suspect that someone is simply abusing notation, and we really do want $|x|rightarrowinfty$.
    $endgroup$
    – ItsJustLogicBro
    Jan 6 at 4:06
















1












$begingroup$



Let $f: mathbb{R}^{m}to mathbb{R}^{n}$ be a continuous function. Prove that the following statements are equivalents:



(i) $displaystylelim_{x to infty}f(x)=infty$;



(ii) $f^{-1}(K)$ is compact for every compact $K subset mathbb{R}^{n}$.




My attempt.



(i) $Longrightarrow$ (ii): If $K subset mathbb{R}^{n}$ is compact, $f^{-1}(K)$ is closed, since $f$ is continuous. If $f^{-1}(K)$ is unbounded, there is a sequence $(x_{n}) subset f^{-1}(K)$ such that $x_{n} to infty$.



I want to conclude that $f(x_{n}) to infty$. How can I ensure it?



(ii) $Longrightarrow$ (i): I dont have a good idea. Can someone help me? I want just a hint, not a solution.










share|cite|improve this question









$endgroup$












  • $begingroup$
    How do you define $xtoinfty$ exactly? With the one-point compactification?
    $endgroup$
    – SmileyCraft
    Jan 6 at 3:55








  • 1




    $begingroup$
    Consider the function $f(x)=e^x$ and the compact set $K=[-1,1]$. Then $lim f(x)=infty$ but $f^{-1}(K)=(-infty,0]$. So either this problem isn't correct or I am losing my mind.
    $endgroup$
    – ItsJustLogicBro
    Jan 6 at 3:58












  • $begingroup$
    @SmileyCraft, my book has not yet exactly defined $x to infty$. When we work with infinite limits, we use the standard definition.
    $endgroup$
    – Lucas Corrêa
    Jan 6 at 4:02










  • $begingroup$
    @ItsJustLogicBro $limlimits_{x to -infty } e^x =0$. So this isn’t a counterexample.
    $endgroup$
    – mathcounterexamples.net
    Jan 6 at 4:02






  • 1




    $begingroup$
    @LucasCorrêa I suspect that someone is simply abusing notation, and we really do want $|x|rightarrowinfty$.
    $endgroup$
    – ItsJustLogicBro
    Jan 6 at 4:06














1












1








1





$begingroup$



Let $f: mathbb{R}^{m}to mathbb{R}^{n}$ be a continuous function. Prove that the following statements are equivalents:



(i) $displaystylelim_{x to infty}f(x)=infty$;



(ii) $f^{-1}(K)$ is compact for every compact $K subset mathbb{R}^{n}$.




My attempt.



(i) $Longrightarrow$ (ii): If $K subset mathbb{R}^{n}$ is compact, $f^{-1}(K)$ is closed, since $f$ is continuous. If $f^{-1}(K)$ is unbounded, there is a sequence $(x_{n}) subset f^{-1}(K)$ such that $x_{n} to infty$.



I want to conclude that $f(x_{n}) to infty$. How can I ensure it?



(ii) $Longrightarrow$ (i): I dont have a good idea. Can someone help me? I want just a hint, not a solution.










share|cite|improve this question









$endgroup$





Let $f: mathbb{R}^{m}to mathbb{R}^{n}$ be a continuous function. Prove that the following statements are equivalents:



(i) $displaystylelim_{x to infty}f(x)=infty$;



(ii) $f^{-1}(K)$ is compact for every compact $K subset mathbb{R}^{n}$.




My attempt.



(i) $Longrightarrow$ (ii): If $K subset mathbb{R}^{n}$ is compact, $f^{-1}(K)$ is closed, since $f$ is continuous. If $f^{-1}(K)$ is unbounded, there is a sequence $(x_{n}) subset f^{-1}(K)$ such that $x_{n} to infty$.



I want to conclude that $f(x_{n}) to infty$. How can I ensure it?



(ii) $Longrightarrow$ (i): I dont have a good idea. Can someone help me? I want just a hint, not a solution.







real-analysis continuity compactness






share|cite|improve this question













share|cite|improve this question











share|cite|improve this question




share|cite|improve this question










asked Jan 6 at 3:49









Lucas CorrêaLucas Corrêa

1,5351421




1,5351421












  • $begingroup$
    How do you define $xtoinfty$ exactly? With the one-point compactification?
    $endgroup$
    – SmileyCraft
    Jan 6 at 3:55








  • 1




    $begingroup$
    Consider the function $f(x)=e^x$ and the compact set $K=[-1,1]$. Then $lim f(x)=infty$ but $f^{-1}(K)=(-infty,0]$. So either this problem isn't correct or I am losing my mind.
    $endgroup$
    – ItsJustLogicBro
    Jan 6 at 3:58












  • $begingroup$
    @SmileyCraft, my book has not yet exactly defined $x to infty$. When we work with infinite limits, we use the standard definition.
    $endgroup$
    – Lucas Corrêa
    Jan 6 at 4:02










  • $begingroup$
    @ItsJustLogicBro $limlimits_{x to -infty } e^x =0$. So this isn’t a counterexample.
    $endgroup$
    – mathcounterexamples.net
    Jan 6 at 4:02






  • 1




    $begingroup$
    @LucasCorrêa I suspect that someone is simply abusing notation, and we really do want $|x|rightarrowinfty$.
    $endgroup$
    – ItsJustLogicBro
    Jan 6 at 4:06


















  • $begingroup$
    How do you define $xtoinfty$ exactly? With the one-point compactification?
    $endgroup$
    – SmileyCraft
    Jan 6 at 3:55








  • 1




    $begingroup$
    Consider the function $f(x)=e^x$ and the compact set $K=[-1,1]$. Then $lim f(x)=infty$ but $f^{-1}(K)=(-infty,0]$. So either this problem isn't correct or I am losing my mind.
    $endgroup$
    – ItsJustLogicBro
    Jan 6 at 3:58












  • $begingroup$
    @SmileyCraft, my book has not yet exactly defined $x to infty$. When we work with infinite limits, we use the standard definition.
    $endgroup$
    – Lucas Corrêa
    Jan 6 at 4:02










  • $begingroup$
    @ItsJustLogicBro $limlimits_{x to -infty } e^x =0$. So this isn’t a counterexample.
    $endgroup$
    – mathcounterexamples.net
    Jan 6 at 4:02






  • 1




    $begingroup$
    @LucasCorrêa I suspect that someone is simply abusing notation, and we really do want $|x|rightarrowinfty$.
    $endgroup$
    – ItsJustLogicBro
    Jan 6 at 4:06
















$begingroup$
How do you define $xtoinfty$ exactly? With the one-point compactification?
$endgroup$
– SmileyCraft
Jan 6 at 3:55






$begingroup$
How do you define $xtoinfty$ exactly? With the one-point compactification?
$endgroup$
– SmileyCraft
Jan 6 at 3:55






1




1




$begingroup$
Consider the function $f(x)=e^x$ and the compact set $K=[-1,1]$. Then $lim f(x)=infty$ but $f^{-1}(K)=(-infty,0]$. So either this problem isn't correct or I am losing my mind.
$endgroup$
– ItsJustLogicBro
Jan 6 at 3:58






$begingroup$
Consider the function $f(x)=e^x$ and the compact set $K=[-1,1]$. Then $lim f(x)=infty$ but $f^{-1}(K)=(-infty,0]$. So either this problem isn't correct or I am losing my mind.
$endgroup$
– ItsJustLogicBro
Jan 6 at 3:58














$begingroup$
@SmileyCraft, my book has not yet exactly defined $x to infty$. When we work with infinite limits, we use the standard definition.
$endgroup$
– Lucas Corrêa
Jan 6 at 4:02




$begingroup$
@SmileyCraft, my book has not yet exactly defined $x to infty$. When we work with infinite limits, we use the standard definition.
$endgroup$
– Lucas Corrêa
Jan 6 at 4:02












$begingroup$
@ItsJustLogicBro $limlimits_{x to -infty } e^x =0$. So this isn’t a counterexample.
$endgroup$
– mathcounterexamples.net
Jan 6 at 4:02




$begingroup$
@ItsJustLogicBro $limlimits_{x to -infty } e^x =0$. So this isn’t a counterexample.
$endgroup$
– mathcounterexamples.net
Jan 6 at 4:02




1




1




$begingroup$
@LucasCorrêa I suspect that someone is simply abusing notation, and we really do want $|x|rightarrowinfty$.
$endgroup$
– ItsJustLogicBro
Jan 6 at 4:06




$begingroup$
@LucasCorrêa I suspect that someone is simply abusing notation, and we really do want $|x|rightarrowinfty$.
$endgroup$
– ItsJustLogicBro
Jan 6 at 4:06










3 Answers
3






active

oldest

votes


















2












$begingroup$

To complete your argument for $(i)$ $|x_n|rightarrow infty$ implies that $f(x_n)rightarrow infty$ by hypothesis.



ii) implies i)Suppose that there exists a sequence $x_n$ such that $lim_n|x_n|=infty$ and $f(x_n)$ is bounded, $f(x_n)$ is contained in a closed ball $B$ which is compact, so $f^{-1}(B)=C$ is compact, and bounded by hypothesis, but $x_nin C$ contradiction since $x_n$ is not bounded.






share|cite|improve this answer











$endgroup$





















    1












    $begingroup$

    The definition of $x_ntoinfty$ I am used to is that for every compact set $K$ there exists $N$ such that for all $n>N$ we have $x_nnotin K$. So then the statement $$lim_{xtoinfty}f(x)=infty$$ would literally mean that for every compact set $K$ there exists compact set $L$ such that for all $xnotin L$ we have $f(x)notin K$, so $f^{-1}(K)subseteq L$.



    I would like to mention how this elegant definition can be made precise with topology. In topology we say that $x_nto x$ if for every open set $U$ containing $x$ there exists $N$ such that for all $n>N$ we have $x_nin U$. There is also a thing in topology called the one-point compactification of a topological space $X$. The idea is to add a "point at infinity", so you get $Xcup{infty}$, and the open sets containing $infty$ are exactly the complements of the compact sets in $X$.






    share|cite|improve this answer









    $endgroup$













    • $begingroup$
      Its a very clever proof for bounded! I didnt know this definition, is that a topological property?
      $endgroup$
      – Lucas Corrêa
      Jan 6 at 12:58



















    1












    $begingroup$

    Regarding (ii) $Longrightarrow$ (i)



    Take $p in mathbb N$. You want to find $M>0$ such that for $Vert x Vert >M$ you have $Vert f(x) Vert >p$.



    The closed ball $K_p$centered on zero with radius $p$ is a compact subset of $mathbb R^n$. $f^{-1}(mathbb R^n setminus K_p) = f^{-1}(mathbb R^n)setminus f^{-1}(K_p)$. And $f^{-1}(K_p)$ is by hypothesis a compact subset of $mathbb R^n$, therefore bounded and included in a closed ball centered on zero with a radius $M>0$.



    Where are done: the image of a point $x$ with a norm $>M$ is such that $Vert f(x)Vert >p$.






    share|cite|improve this answer











    $endgroup$













      Your Answer





      StackExchange.ifUsing("editor", function () {
      return StackExchange.using("mathjaxEditing", function () {
      StackExchange.MarkdownEditor.creationCallbacks.add(function (editor, postfix) {
      StackExchange.mathjaxEditing.prepareWmdForMathJax(editor, postfix, [["$", "$"], ["\\(","\\)"]]);
      });
      });
      }, "mathjax-editing");

      StackExchange.ready(function() {
      var channelOptions = {
      tags: "".split(" "),
      id: "69"
      };
      initTagRenderer("".split(" "), "".split(" "), channelOptions);

      StackExchange.using("externalEditor", function() {
      // Have to fire editor after snippets, if snippets enabled
      if (StackExchange.settings.snippets.snippetsEnabled) {
      StackExchange.using("snippets", function() {
      createEditor();
      });
      }
      else {
      createEditor();
      }
      });

      function createEditor() {
      StackExchange.prepareEditor({
      heartbeatType: 'answer',
      autoActivateHeartbeat: false,
      convertImagesToLinks: true,
      noModals: true,
      showLowRepImageUploadWarning: true,
      reputationToPostImages: 10,
      bindNavPrevention: true,
      postfix: "",
      imageUploader: {
      brandingHtml: "Powered by u003ca class="icon-imgur-white" href="https://imgur.com/"u003eu003c/au003e",
      contentPolicyHtml: "User contributions licensed under u003ca href="https://creativecommons.org/licenses/by-sa/3.0/"u003ecc by-sa 3.0 with attribution requiredu003c/au003e u003ca href="https://stackoverflow.com/legal/content-policy"u003e(content policy)u003c/au003e",
      allowUrls: true
      },
      noCode: true, onDemand: true,
      discardSelector: ".discard-answer"
      ,immediatelyShowMarkdownHelp:true
      });


      }
      });














      draft saved

      draft discarded


















      StackExchange.ready(
      function () {
      StackExchange.openid.initPostLogin('.new-post-login', 'https%3a%2f%2fmath.stackexchange.com%2fquestions%2f3063461%2fif-f-is-continuous-displaystyle-lim-x-to-inftyfx-infty-iff-f-1k%23new-answer', 'question_page');
      }
      );

      Post as a guest















      Required, but never shown

























      3 Answers
      3






      active

      oldest

      votes








      3 Answers
      3






      active

      oldest

      votes









      active

      oldest

      votes






      active

      oldest

      votes









      2












      $begingroup$

      To complete your argument for $(i)$ $|x_n|rightarrow infty$ implies that $f(x_n)rightarrow infty$ by hypothesis.



      ii) implies i)Suppose that there exists a sequence $x_n$ such that $lim_n|x_n|=infty$ and $f(x_n)$ is bounded, $f(x_n)$ is contained in a closed ball $B$ which is compact, so $f^{-1}(B)=C$ is compact, and bounded by hypothesis, but $x_nin C$ contradiction since $x_n$ is not bounded.






      share|cite|improve this answer











      $endgroup$


















        2












        $begingroup$

        To complete your argument for $(i)$ $|x_n|rightarrow infty$ implies that $f(x_n)rightarrow infty$ by hypothesis.



        ii) implies i)Suppose that there exists a sequence $x_n$ such that $lim_n|x_n|=infty$ and $f(x_n)$ is bounded, $f(x_n)$ is contained in a closed ball $B$ which is compact, so $f^{-1}(B)=C$ is compact, and bounded by hypothesis, but $x_nin C$ contradiction since $x_n$ is not bounded.






        share|cite|improve this answer











        $endgroup$
















          2












          2








          2





          $begingroup$

          To complete your argument for $(i)$ $|x_n|rightarrow infty$ implies that $f(x_n)rightarrow infty$ by hypothesis.



          ii) implies i)Suppose that there exists a sequence $x_n$ such that $lim_n|x_n|=infty$ and $f(x_n)$ is bounded, $f(x_n)$ is contained in a closed ball $B$ which is compact, so $f^{-1}(B)=C$ is compact, and bounded by hypothesis, but $x_nin C$ contradiction since $x_n$ is not bounded.






          share|cite|improve this answer











          $endgroup$



          To complete your argument for $(i)$ $|x_n|rightarrow infty$ implies that $f(x_n)rightarrow infty$ by hypothesis.



          ii) implies i)Suppose that there exists a sequence $x_n$ such that $lim_n|x_n|=infty$ and $f(x_n)$ is bounded, $f(x_n)$ is contained in a closed ball $B$ which is compact, so $f^{-1}(B)=C$ is compact, and bounded by hypothesis, but $x_nin C$ contradiction since $x_n$ is not bounded.







          share|cite|improve this answer














          share|cite|improve this answer



          share|cite|improve this answer








          edited Jan 6 at 4:05

























          answered Jan 6 at 3:59









          Tsemo AristideTsemo Aristide

          59.6k11446




          59.6k11446























              1












              $begingroup$

              The definition of $x_ntoinfty$ I am used to is that for every compact set $K$ there exists $N$ such that for all $n>N$ we have $x_nnotin K$. So then the statement $$lim_{xtoinfty}f(x)=infty$$ would literally mean that for every compact set $K$ there exists compact set $L$ such that for all $xnotin L$ we have $f(x)notin K$, so $f^{-1}(K)subseteq L$.



              I would like to mention how this elegant definition can be made precise with topology. In topology we say that $x_nto x$ if for every open set $U$ containing $x$ there exists $N$ such that for all $n>N$ we have $x_nin U$. There is also a thing in topology called the one-point compactification of a topological space $X$. The idea is to add a "point at infinity", so you get $Xcup{infty}$, and the open sets containing $infty$ are exactly the complements of the compact sets in $X$.






              share|cite|improve this answer









              $endgroup$













              • $begingroup$
                Its a very clever proof for bounded! I didnt know this definition, is that a topological property?
                $endgroup$
                – Lucas Corrêa
                Jan 6 at 12:58
















              1












              $begingroup$

              The definition of $x_ntoinfty$ I am used to is that for every compact set $K$ there exists $N$ such that for all $n>N$ we have $x_nnotin K$. So then the statement $$lim_{xtoinfty}f(x)=infty$$ would literally mean that for every compact set $K$ there exists compact set $L$ such that for all $xnotin L$ we have $f(x)notin K$, so $f^{-1}(K)subseteq L$.



              I would like to mention how this elegant definition can be made precise with topology. In topology we say that $x_nto x$ if for every open set $U$ containing $x$ there exists $N$ such that for all $n>N$ we have $x_nin U$. There is also a thing in topology called the one-point compactification of a topological space $X$. The idea is to add a "point at infinity", so you get $Xcup{infty}$, and the open sets containing $infty$ are exactly the complements of the compact sets in $X$.






              share|cite|improve this answer









              $endgroup$













              • $begingroup$
                Its a very clever proof for bounded! I didnt know this definition, is that a topological property?
                $endgroup$
                – Lucas Corrêa
                Jan 6 at 12:58














              1












              1








              1





              $begingroup$

              The definition of $x_ntoinfty$ I am used to is that for every compact set $K$ there exists $N$ such that for all $n>N$ we have $x_nnotin K$. So then the statement $$lim_{xtoinfty}f(x)=infty$$ would literally mean that for every compact set $K$ there exists compact set $L$ such that for all $xnotin L$ we have $f(x)notin K$, so $f^{-1}(K)subseteq L$.



              I would like to mention how this elegant definition can be made precise with topology. In topology we say that $x_nto x$ if for every open set $U$ containing $x$ there exists $N$ such that for all $n>N$ we have $x_nin U$. There is also a thing in topology called the one-point compactification of a topological space $X$. The idea is to add a "point at infinity", so you get $Xcup{infty}$, and the open sets containing $infty$ are exactly the complements of the compact sets in $X$.






              share|cite|improve this answer









              $endgroup$



              The definition of $x_ntoinfty$ I am used to is that for every compact set $K$ there exists $N$ such that for all $n>N$ we have $x_nnotin K$. So then the statement $$lim_{xtoinfty}f(x)=infty$$ would literally mean that for every compact set $K$ there exists compact set $L$ such that for all $xnotin L$ we have $f(x)notin K$, so $f^{-1}(K)subseteq L$.



              I would like to mention how this elegant definition can be made precise with topology. In topology we say that $x_nto x$ if for every open set $U$ containing $x$ there exists $N$ such that for all $n>N$ we have $x_nin U$. There is also a thing in topology called the one-point compactification of a topological space $X$. The idea is to add a "point at infinity", so you get $Xcup{infty}$, and the open sets containing $infty$ are exactly the complements of the compact sets in $X$.







              share|cite|improve this answer












              share|cite|improve this answer



              share|cite|improve this answer










              answered Jan 6 at 4:18









              SmileyCraftSmileyCraft

              3,749519




              3,749519












              • $begingroup$
                Its a very clever proof for bounded! I didnt know this definition, is that a topological property?
                $endgroup$
                – Lucas Corrêa
                Jan 6 at 12:58


















              • $begingroup$
                Its a very clever proof for bounded! I didnt know this definition, is that a topological property?
                $endgroup$
                – Lucas Corrêa
                Jan 6 at 12:58
















              $begingroup$
              Its a very clever proof for bounded! I didnt know this definition, is that a topological property?
              $endgroup$
              – Lucas Corrêa
              Jan 6 at 12:58




              $begingroup$
              Its a very clever proof for bounded! I didnt know this definition, is that a topological property?
              $endgroup$
              – Lucas Corrêa
              Jan 6 at 12:58











              1












              $begingroup$

              Regarding (ii) $Longrightarrow$ (i)



              Take $p in mathbb N$. You want to find $M>0$ such that for $Vert x Vert >M$ you have $Vert f(x) Vert >p$.



              The closed ball $K_p$centered on zero with radius $p$ is a compact subset of $mathbb R^n$. $f^{-1}(mathbb R^n setminus K_p) = f^{-1}(mathbb R^n)setminus f^{-1}(K_p)$. And $f^{-1}(K_p)$ is by hypothesis a compact subset of $mathbb R^n$, therefore bounded and included in a closed ball centered on zero with a radius $M>0$.



              Where are done: the image of a point $x$ with a norm $>M$ is such that $Vert f(x)Vert >p$.






              share|cite|improve this answer











              $endgroup$


















                1












                $begingroup$

                Regarding (ii) $Longrightarrow$ (i)



                Take $p in mathbb N$. You want to find $M>0$ such that for $Vert x Vert >M$ you have $Vert f(x) Vert >p$.



                The closed ball $K_p$centered on zero with radius $p$ is a compact subset of $mathbb R^n$. $f^{-1}(mathbb R^n setminus K_p) = f^{-1}(mathbb R^n)setminus f^{-1}(K_p)$. And $f^{-1}(K_p)$ is by hypothesis a compact subset of $mathbb R^n$, therefore bounded and included in a closed ball centered on zero with a radius $M>0$.



                Where are done: the image of a point $x$ with a norm $>M$ is such that $Vert f(x)Vert >p$.






                share|cite|improve this answer











                $endgroup$
















                  1












                  1








                  1





                  $begingroup$

                  Regarding (ii) $Longrightarrow$ (i)



                  Take $p in mathbb N$. You want to find $M>0$ such that for $Vert x Vert >M$ you have $Vert f(x) Vert >p$.



                  The closed ball $K_p$centered on zero with radius $p$ is a compact subset of $mathbb R^n$. $f^{-1}(mathbb R^n setminus K_p) = f^{-1}(mathbb R^n)setminus f^{-1}(K_p)$. And $f^{-1}(K_p)$ is by hypothesis a compact subset of $mathbb R^n$, therefore bounded and included in a closed ball centered on zero with a radius $M>0$.



                  Where are done: the image of a point $x$ with a norm $>M$ is such that $Vert f(x)Vert >p$.






                  share|cite|improve this answer











                  $endgroup$



                  Regarding (ii) $Longrightarrow$ (i)



                  Take $p in mathbb N$. You want to find $M>0$ such that for $Vert x Vert >M$ you have $Vert f(x) Vert >p$.



                  The closed ball $K_p$centered on zero with radius $p$ is a compact subset of $mathbb R^n$. $f^{-1}(mathbb R^n setminus K_p) = f^{-1}(mathbb R^n)setminus f^{-1}(K_p)$. And $f^{-1}(K_p)$ is by hypothesis a compact subset of $mathbb R^n$, therefore bounded and included in a closed ball centered on zero with a radius $M>0$.



                  Where are done: the image of a point $x$ with a norm $>M$ is such that $Vert f(x)Vert >p$.







                  share|cite|improve this answer














                  share|cite|improve this answer



                  share|cite|improve this answer








                  edited Jan 6 at 9:09

























                  answered Jan 6 at 4:12









                  mathcounterexamples.netmathcounterexamples.net

                  27k22158




                  27k22158






























                      draft saved

                      draft discarded




















































                      Thanks for contributing an answer to Mathematics Stack Exchange!


                      • Please be sure to answer the question. Provide details and share your research!

                      But avoid



                      • Asking for help, clarification, or responding to other answers.

                      • Making statements based on opinion; back them up with references or personal experience.


                      Use MathJax to format equations. MathJax reference.


                      To learn more, see our tips on writing great answers.




                      draft saved


                      draft discarded














                      StackExchange.ready(
                      function () {
                      StackExchange.openid.initPostLogin('.new-post-login', 'https%3a%2f%2fmath.stackexchange.com%2fquestions%2f3063461%2fif-f-is-continuous-displaystyle-lim-x-to-inftyfx-infty-iff-f-1k%23new-answer', 'question_page');
                      }
                      );

                      Post as a guest















                      Required, but never shown





















































                      Required, but never shown














                      Required, but never shown












                      Required, but never shown







                      Required, but never shown

































                      Required, but never shown














                      Required, but never shown












                      Required, but never shown







                      Required, but never shown







                      Popular posts from this blog

                      Cabo Verde

                      Gyllenstierna

                      Karlovacs län